You are on page 1of 4

Limit Examples using the precise denition of the limit Denition 1 (Limit).

Let f be a function dened on some open interval containing a, except possibly at a itself. lim f (x) = L means for every number
xa

> 0 there is a number > 0 such that

|f (x) L| <

whenever

0 < |x a| < .

1. Prove lim (4x + 7) = 19 .


x3

Solution: We will complete this problem in two steps. Part A. is scratch work and part B. is the proof. A. Given > 0, we need to nd > 0 (in terms of ) so that |(4x + 7) 19| < We begin with the whenever 0 < |x 3| < .

inequality to deduce what must be. |(4x + 7) 19| = |4x 12| = 4|x 3| <

Notice that |(4x + 7) 19| < B. Proof. Fix

when |x 3| < 4 , so we nd = 4 .

> 0 and let = 4 . If 0 < |x 3| < , then |(4x + 7) 19| = |4x 12| = 4|x 3| < 4 = 4 4 = .

Therefore, we have shown lim (4x + 7) = 19 , since


x3

|(4x + 7) 19| <

whenever 0 < |x 3| < .

2. Prove lim (x2 +2x1) = 14 .


x3

Solution: As before, we will proceed in two steps. A. Given > 0, we need to nd > 0 (in terms of ) so that |(x2 + 2x 1) 14| < We begin with the whenever 0 < |x 3| < .

inequality to deduce what must be. |(x2 + 2x 1) 14| = |x2 + 2x 15| = |(x + 5)(x 3)| = |x + 5||x 3| <

Remember, our goal is to nd , i.e., an upper bound on |x 3|, in terms of . In the rst example we arrived at a similar situation. In that problem we divided by 4 to get = /4. In this problem we want to perform a similar operation, that is, divide by |x + 5|, but we want our delta only in terms of , not x. We achieve this as follows. If we can nd a number C so that |x + 5| < C , then certainly |x + 5||x 3| < C |x 3| which we can make smaller than by taking |x 3| < /C , that is, we divide by the bound on |x + 5| rather than |x + 5|. Now we need to nd C . Since we are only interested in values of x that are close to 3, we can restrict our attention to those values of x which satisfy 2 < x < 4. This is equivalent to 7 < x + 5 < 9, so we see that restricting our attention to the x-interval (2, 4), we obtain |x + 5| < 9, so we let C = 9. Then we see that |x 3| < 9 . So we can now pick . = min 1, 9

We need to use this . Remember that we found C = 9 by assuming that we are in the x-interval (2, 4), i.e. |x 3| < 1. From this we nd that we also desire |x 3| < /9. Since we need to satisfy two conditions, we pick the smaller to satisfy both. B. Proof. Fix > 0 and let = min{1, /9}. If 0 < |x 3| < , then |x 3| < 1 so that |x + 5| < 9. Also, if 0 < |x 3| < , then 0 < |x 3| < /9 so we have |(x2 + 2x 1) 14| = |x2 + 2x 15| = |(x + 5)(x 3)| = |x + 5||x 3| < 9 9 =

Denition 2 (Left-Hand Limit).


xa

lim f (x) = L

means for every number

> 0 there is a number > 0 such that |f (x) L| < whenever a < x < a.

Denition 3 (Right-Hand Limit).


xa+

lim f (x) = L

means for every number

> 0 there is a number > 0 such that |f (x) L| < whenever a < x < a + .

Denition 4 (Innite Limit).


xa

lim f (x) =

means for every number M > 0 there is a number > 0 such that f (x) > M whenever 0 < |x a| < .

Denition 5 (Innite Limit).


xa

lim f (x) =

means for every number N < 0 there is a number > 0 such that f (x) < N whenever 0 < |x a| < .

Notice in Denition 1, that we require 0 < |x a| < . This is equivalent to a < x < a + . The left-hand and right-hand limit denitions are the same as Denition 1 except that as we are only interested in one side of a, we only consider one part of this compound inequality. For the innite limit we know that f becomes arbitrarily large, that is, if we pick any number M , we can always pick an x close to a such that f (x) is bigger than M .

3. Prove

x2+

lim

1 = . x2

Solution: As before, we will proceed in two steps. A. Given M > 0, we need to nd > 0 so that 1 >M x2 whenever 2 < x < 2 + .

We begin with the M inequality to deduce what must be. > M 1 0 < x2 < M 1 2 < x < 2+ M So we see that we want = B. Proof. Fix > 0 and let =
1 M. 1 M. 1 x2

If 2 < x < 2 + 2 < 0 <

1 M,

then

1 x < 2+ M 1 x2 < M 1 > M x2

Therefore, we have shown that

x2+

lim

1 = , since x2

1 >M x2

whenever 2 < x < 2 + .

You might also like